1. Soient \(a_1,\dots, a_n,b_1,\dots,b_n\) des réels strictement positifs, montrer que \[\max\left\lbrace \dfrac{a_1}{b_1}+\dfrac{a_2}{b_2}+\dots+\dfrac{a_n}{b_n},\ \dfrac{b_1}{a_1}+\dfrac{b_2}{a_2}+\dots+\dfrac{b_n}{a_n}\right\rbrace \geq n.\]

  2. Soient \(a_1,\dots, a_n\) des réels \(\geq 1\), montrer que \[(1+a_1)(1+a_2)\dots(1+a_n)\geq \dfrac{2^n}{n+1}(1+a_1+a_2+\dots+a_n).\]


Barre utilisateur

[ID: 2529] [Date de publication: 9 novembre 2022 11:18] [Catégorie(s): Géométrie ] [ Nombre commentaires: 1] [nombre d'éditeurs: 1 ] [Editeur(s): Emmanuel Vieillard-Baron ] [nombre d'auteurs: 1 ] [Auteur(s): Patrice Lassère ]




Solution(s)

Solution(s)

Deux inégalités
Par Patrice Lassère le 9 novembre 2022 11:18
  1.  - Première solution : Avec l’inégalité arithmético-géométrique1 on a \[A:=\left( \dfrac{a_1a_2\dots a_n}{b_1b_2\dots b_n}\right)^{1/n}\leq \dfrac{1}{n}\left( \dfrac{a_1}{b_1}+\dfrac{a_2}{b_2}+\dots+\dfrac{a_n}{b_n}\right)\] et \[A^{-1}=\left( \dfrac{b_1b_2\dots b_n}{a_1a_2\dots a_n}\right)^{1/n}\leq \dfrac{1}{n}\left( \dfrac{b_1}{a_1}+\dfrac{b_2}{a_2}+\dots+\dfrac{b_n}{a_n}\right).\] Comme l’un des réels \(A\) et \(A^{-1}\) est nécessairement supérieur ou égal à \(1\) le résultat suit.

    Seconde solution : Avec l’inégalité de Cauchy-Schwarz : \[\left( \sum_{i=1}^n\dfrac{a_i}{b_i}\right) \cdot \left( \sum_{i=1}^n\dfrac{b_i}{a_i}\right)\geq n^2,\] par conséquent dans le terme de gauche, l’un des deux facteurs se doit d’être supérieur ou égal à \(n\).

    Troisième solution : Par convexité sur \(\mathbb R_+^\star\) de l’application \(f\ :\ x\mapsto f(x)=x^{-1}\) nous avons \[f\left( \dfrac{1}{n}\sum_{i=1}^n\dfrac{a_i}{b_i}\right) \leq \dfrac{1}{n}\sum_{i=1}^n f\left(\dfrac{a_i}{b_i}\right)\] soit \[n^2\leq \left( \sum_{i=1}^n\dfrac{a_i}{b_i}\right) \cdot \left( \sum_{i=1}^n\dfrac{b_i}{a_i}\right)\] et on retrouve l’inégalité de la seconde solution.

  2.  - Première solution : Par récurrence sur \(n\geq 1\) : c’est clair pour \(n=1\) ; supposons la formule valide jusqu’au rang \(n\), alors \[\begin{aligned} (1+a_1)(1+a_2)\dots(1+a_n)(1+a_{n+1})&\geq \dfrac{2^n}{n+1}(1+a_1+a_2+\dots+a_n)(1+a_{n+1})\\ &:=\dfrac{2^n}{n+1}(1+a)(1+b), \end{aligned}\] (avec \(a=a_1+a_2+\dots+a_n, b=a_{n+1}\)) et il ne reste plus qu’à montrer \[\dfrac{2^n}{n+1}(1+a)(1+b) \geq \dfrac{2^{n+1}}{n+2}(1+a+b).\] Cette inégalité équivaut à \[(n+2)(1+a)(1+b)\geq 2(n+1)(1+a+b).\] Mais \[(n+2)(1+a)(1+b)- 2(n+1)(1+a+b)=2(ab-n)+n(a-1)(b-1),\] et les deux derniers termes sont positifs car \(a=a_1+a_2+\dots+a_n\geq n\) et \(b=a_{n+1}\geq 1\).

    Seconde solution : L’inégalité proposée équivaut à \[\left( \dfrac{1+a_1}{2}\right)\left( \dfrac{1+a_2}{2}\right)\dots \left( \dfrac{1+a_n}{2}\right)\geq\dfrac{1+a_1+a_2+\dots+a_n}{n+1}\] ou encore \[(1+x_1)(1+x_2)\dots(1+x_n)\geq 1+\dfrac{2}{n+1}(x_1+x_2+\dots+x_n)\] en posant \(x_i=(a_i-1)/2,\ (1\leq i\leq n)\). Mais par positivité des \(x_i\) \[\begin{aligned}(1+x_1)(1+x_2)\dots(1+x_n)&\geq 1+x_1+x_2+\dots+x_n\\ &\geq 1+\dfrac{2}{n+1}(x_1+x_2+\dots+x_n) \end{aligned}\]

    CQFD.


  1. 1  L’inégalité arithmético-géométrique : pour toute suite de réels positifs \(a_1,a_2,\dots,a_n\) on a \((a_1a_2\dots a_n)^{1/n}\leq \frac{a_1+a_2+\dots+a_n}{n}\)

Documents à télécharger